4
$\begingroup$

I will start with a definition.

A monotone function $f$ on $[a,b]$ is called singular if $f'=0$ almost everywhere.

Let $f$ be a nondecreasing function on $[a,b]$ such that given $\epsilon~,~\delta\gt 0$, $\exists$ a finite collection $\{[y_k,x_k]\}$ of nonoverlapping intervals such that $$\sum |x_k-y_k|\lt \delta~~~~~\text{and}~~~~\sum\left(f(x_k)-f(y_k)\right)\gt f(b)-f(a)-\epsilon.$$
I would like to show that $f$ is singular.

Attempt:

From a previous exercise, I showed that a monotone function $f$ can be written as the sum of an absolutely continuous function, $g$ and a singular function, $h$. Thus $$ f = g + h ,~~~\text{where}~~g=\int_a^x f' .$$
My goal is to show that $g=0$ almost everywhere. Let $I=\bigcup (y_k,x_k).$ Then $$ \int_I f' = \sum \int_{(x_k,y_k)}~f' = \sum\left(f(x_k)-f(y_k)\right)\lt \epsilon.$$
Now, I know that since $f'$ is integrable, there is an $\epsilon$ such that $$\int_{[a,b]\setminus I}~f'\lt \epsilon. $$ But $$\begin{align*} 0\leq \int_a^b f' & = \int_I f'+\int_{[a,b]\setminus I}~f'\\ & \lt \epsilon + \epsilon\\ & = 2\epsilon. \end{align*}$$
Since $\epsilon$ is arbitrary, we can let $\epsilon \rightarrow 0$ and thus $$ \int_a^b f'=0 ~~\text{and }~~g = 0.$$

Is what I've done right? Is there another way of approaching the problem?
Thanks.

  • 0
    What are $(\alpha_i,\beta_i)$?2011-11-05
  • 0
    $ \int_{(x_k,y_k)}~f' = f(x_k)-f(y_k)$ is incorrect; is this a typo? In any case, it is true that $\int_I f'$ can be made arbitrarily small, but this is by choosing $\delta$ small enough and using integrability of $f'$; I do not see justification in your solution.2011-11-05
  • 0
    $\bigcup(\alpha_i,\beta_i)=[a,b]\setminus I$...so how do I justify that there is such a $\delta$?2011-11-05
  • 0
    $[a,b]\setminus I$ will not be a union of open intervals. (So you could just leave it as $[a,b]\setminus I$ or give it a shorter name if you want.) As for the question in your comment, see http://math.stackexchange.com/questions/40384/showing-uniform-continuity.2011-11-05
  • 0
    okay. Thanks for the link.2011-11-05
  • 0
    There are at least 2 problems where you wrote "$\int_I f' = \sum \int_{(x_k,y_k)}~f' = \sum\left(f(x_k)-f(y_k)\right)\lt \epsilon.$". One problem is that $\int_{(x_k,y_k)}f' = g(y_k)-g(x_k)$, not $f(x_k)-f(y_k)$. Another is that whether or not the integral is less than $\varepsilon$ depends on $I$. The $\varepsilon$ must be given first, and justification must be given for choosing an $I$ such that the integral is less than $\varepsilon$. (Details for one approach to this are given in t.b.'s answer at the linked question.)2011-11-06
  • 0
    The next statement is not helpful: "...there is an $\varepsilon$ such that...". That is true, but this will have no connection to other parts of your argument, such as where you later say that $\varepsilon$ is arbitrary. Since $\varepsilon$ must be arbitrary, you would have to *show* that $\int_I f'<\varepsilon$ if you are going to use that fact (and it will rely on how $I$ is chosen).2011-11-06
  • 0
    (Oops, I meant $\int_{[a,b]\setminus I}f'<\varepsilon$ in my last comment.)2011-11-07

1 Answers 1

2

Given $\varepsilon>0$, there is a $\delta>0$ such that $\int_I f'<\varepsilon$ whenever $m(I)<\delta$. Thus you can choose $I=\cup(x_k,y_k)$ such that $\int_I f'<\varepsilon$ and $\sum_k f(y_k)-f(x_k)>f(b)-f(a)-\varepsilon$.

This implies that

$\begin{align*} &f(b)-f(a)-(h(b)-h(a))\\ &\leq f(b)-f(a) - (\sum_k h(y_k)-h(x_k))\\ &<\sum_k(g(y_k)-g(x_k))+\varepsilon\\ &=\int_I f' +\varepsilon\\ &<2\varepsilon. \end{align*}$

Since $\varepsilon$ was arbitrary, this implies that $f(b)-f(a)=h(b)-h(a)$, so $g(b)-g(a)=0$, which in turn implies that $g=0$ everywhere. (Note how monotonicity is used here several times.)

  • 0
    This may seem trivial, but why is the first inequality true? Everything else is fine. Thanks.2011-11-14
  • 0
    Assume that there are $N$ intervals arranged so that $x_1. Then $(h(y_1)-h(x_1))+(h(y_2)-h(x_2))+\cdots+(h(y_N)-h(x_N)) $ $=h(y_N)+(h(y_{N-1})-h(x_N))+\cdots+(h(y_1)-h(x_2))-h(x_1)$ $\leq h(y_N)-h(x_1)\leq h(b)-h(a)$ because $h$ is increasing. Perhaps more clearly (but less precisely), $h(b)-h(a)$ gives the total amount that $h$ changes, whereas $\sum_k h(y_k)-h(x_k)$ only counts the change that $h$ does on some nonoverlapping subintervals of $[a,b]$.2011-11-14
  • 0
    oh ok....Thanks.2011-11-15